Verdland

This topic has expert replies
Master | Next Rank: 500 Posts
Posts: 202
Joined: Tue Sep 08, 2009 11:34 pm
Thanked: 15 times
GMAT Score:760

Verdland

by cbenk121 » Wed Nov 11, 2009 7:34 pm
Radio stations with radio data system (RDS) technology broadcast special program information that only radios with an RDS feature can receive. Between 1994 and 1996, the number of RDS radio stations in Verdland increased from 250 to 600. However, since the number of RDS-equipped radios in Verdland was about the same in 1996 as in 1994, the number of Verdlanders received the special program information probably did not increase significantly.

Which of the following is an assumption on which the argument depends?

(A) Few if any of the RDS radio stations that began broadcasting in Verdland after 1994 broadcast to people with RDS-equipped radios living in areas not previously reached by RDS stations.
(B) In 1996 most Verdlanders who lived within the listening area of a RDS station already had a radio equipped to receive RDS.
(C) Equipping a radio station with RDS technology does not decrease the station's listening area.
(D) In 1996 Verdlanders who did not own radios equipped to receive RDS could not receive any programming from the RDS radio stations that began broadcasting in Verdland after 1994.
(E) The RDS radio stations in Verdland in 1996 did not all offer the same type of programming.

OA: A

GMAT Instructor
Posts: 1302
Joined: Mon Oct 19, 2009 2:13 pm
Location: Toronto
Thanked: 539 times
Followed by:164 members
GMAT Score:800

Re: Verdland

by Testluv » Wed Nov 11, 2009 8:28 pm
cbenk121 wrote:Radio stations with radio data system (RDS) technology broadcast special program information that only radios with an RDS feature can receive. Between 1994 and 1996, the number of RDS radio stations in Verdland increased from 250 to 600. However, since the number of RDS-equipped radios in Verdland was about the same in 1996 as in 1994, the number of Verdlanders received the special program information probably did not increase significantly.

Which of the following is an assumption on which the argument depends?

(A) Few if any of the RDS radio stations that began broadcasting in Verdland after 1994 broadcast to people with RDS-equipped radios living in areas not previously reached by RDS stations.
(B) In 1996 most Verdlanders who lived within the listening area of a RDS station already had a radio equipped to receive RDS.
(C) Equipping a radio station with RDS technology does not decrease the station's listening area.
(D) In 1996 Verdlanders who did not own radios equipped to receive RDS could not receive any programming from the RDS radio stations that began broadcasting in Verdland after 1994.
(E) The RDS radio stations in Verdland in 1996 did not all offer the same type of programming.

OA: A
Hi cbenk121,

The number of RDS-equipped radios did not increase. He thinks this means that the number of Verdlanders receiving the special program also has not increased. But he is assuming that ALL of the radios were always receiving radio signals. But just because they are the "only" radios capable of receiving signals does not mean that all of them were receiving signals (back in 1994).

DENIAL TEST: if some of the radios were NOT receiving signals in 1994, but after the increase of radio stations sending these signals, ARE receiving signals in 1996, then MORE people are receiving these signals, and the argument falls apart.

For example:

It could be that in 1994 all 250 of the radio stations were congregated in just a couple of cities on the west coast of Verdland. But some of the people who own the receivers could have been on the East Coast, and so, were too far away to receive signals.

Then, in 1996 there are 600 of these radio stations. Now, there are also radio stations on the East Coast receiving signals. Now, a greater number of Verdlanders are receiving signals, and the argument falls apart.

Strategically: because the conclusion is talking about "not increase" the scope of the argument is whether or not there was an increase. Accordingly, we should be partial to answer choices that bring up the idea of difference and number.
Last edited by Testluv on Wed Nov 11, 2009 10:11 pm, edited 1 time in total.
Kaplan Teacher in Toronto

Master | Next Rank: 500 Posts
Posts: 199
Joined: Sat Oct 24, 2009 4:43 pm
Thanked: 22 times
GMAT Score:710

Re: Verdland

by palvarez » Wed Nov 11, 2009 8:41 pm
cbenk121 wrote:Radio stations with radio data system (RDS) technology broadcast special program information that only radios with an RDS feature can receive. Between 1994 and 1996, the number of RDS radio stations in Verdland increased from 250 to 600. However, since the number of RDS-equipped radios in Verdland was about the same in 1996 as in 1994, the number of Verdlanders received the special program information probably did not increase significantly.

Which of the following is an assumption on which the argument depends?

(A) Few if any of the RDS radio stations that began broadcasting in Verdland after 1994 broadcast to people with RDS-equipped radios living in areas not previously reached by RDS stations.
(B) In 1996 most Verdlanders who lived within the listening area of a RDS station already had a radio equipped to receive RDS.
(C) Equipping a radio station with RDS technology does not decrease the station's listening area.
(D) In 1996 Verdlanders who did not own radios equipped to receive RDS could not receive any programming from the RDS radio stations that began broadcasting in Verdland after 1994.
(E) The RDS radio stations in Verdland in 1996 did not all offer the same type of programming.

OA: A
This is a necessary assumption: so, dont look for any phrase that links premisses to the conclusion. PS calls it defender type. Look for "not", and analyze those choices. A is the fit

Legendary Member
Posts: 869
Joined: Wed Aug 26, 2009 3:49 pm
Location: California
Thanked: 13 times
Followed by:3 members

by heshamelaziry » Wed Nov 11, 2009 9:59 pm
PS says that with a defender you try to

1) eliminate another cause for the effect.

2) Show that when the cause occurs, the effect occurs

3) show that when the cause does not occur, the effect does not occur.

4) show that the relationship is not reversed.

5) show that there nothing wrong with the data given.


How do I apply these to this question? Also, I had difficulty using the denial test with answer choices. In A, there are 2 negations and a one confirmation(if i can call it so)


Please help.

GMAT Instructor
Posts: 1302
Joined: Mon Oct 19, 2009 2:13 pm
Location: Toronto
Thanked: 539 times
Followed by:164 members
GMAT Score:800

by Testluv » Wed Nov 11, 2009 10:13 pm
heshamelaziry wrote:PS says that with a defender you try to

1) eliminate another cause for the effect.

2) Show that when the cause occurs, the effect occurs

3) show that when the cause does not occur, the effect does not occur.

4) show that the relationship is not reversed.

5) show that there nothing wrong with the data given.


How do I apply these to this question? Also, I had difficulty using the denial test with answer choices. In A, there are 2 negations and a one confirmation(if i can call it so)


Please help.
Hi Hesham,

Hi Hesham,

Unfortunately, I can't help you in deciphering the mass of memorized text that PS-trained test-takers seem to bring to every CR question.(!)

What I can tell you is that there is one and only one denial of choice A. You have to negate the meaning of the choice.
Kaplan Teacher in Toronto

Legendary Member
Posts: 869
Joined: Wed Aug 26, 2009 3:49 pm
Location: California
Thanked: 13 times
Followed by:3 members

by heshamelaziry » Wed Nov 11, 2009 10:17 pm
THanks. Is there a way to differentiate between necessary assumption and sufficient assumption ?

GMAT Instructor
Posts: 1302
Joined: Mon Oct 19, 2009 2:13 pm
Location: Toronto
Thanked: 539 times
Followed by:164 members
GMAT Score:800

by Testluv » Wed Nov 11, 2009 10:35 pm
heshamelaziry wrote:THanks. Is there a way to differentiate between necessary assumption and sufficient assumption ?
Hi Hesham,

Of course. It would be unfair of the test-maker to leave it open to interpretation whether the question was asking for a necessary or sufficient assumption.

A question that asks you for an assumption that the argument DEPENDS on, RELIES on or is REQUIRED is a necessary assumption question. We can call these "necessary" words.

Sufficient assumption questions are usually phrased like this: "which of the following, if assumed, would enable/allow the conclusion to follow".

A necessary assumption is something that without it--if you did not have it--the argument would no longer hold (would fall apart). (That's why we can use denial test in necessary assumption).

A sufficient assumption is something that with it--if you had it--it would guarantee that the argument would hold. (That's why denial test isn't very helpful in sufficient assumption.)

And, because necessary assumption questions are so much more common, if you are not sure, then it is probably a necessary assumption question.

Let's consider an example. Here is a sample argument:

"The average 40 year-old North American male will have watched one million TV commercials in his or her lifetime. We may safely conclude, therefore, that the TV commercial has influenced North American habits of thought."

Now, here are two statements:

I Anything people are exposed to ingreat quantity will influence habits of thought.

II Certain forms of communication to which certain people are subjected will affect their habits of thought.

The first statement is a sufficient assumption. If it were true that ANYTHING in great quantity will influence thought, then since the TV commercial counts as "anything," if we plugged this into the argument, then the conclusion certainly follows. But it is not necessary--the argument does not necessarily fall apart without this statement. His argument does not NEED the assumption that ANYTHING in great quantity will influence habits of thought. (This is why extreme is usually wrong in necessary assumption but not in sufficient assumption questions.)

The second statement is a necessary assumption. He HAS to assume that certain (ie, some = at least one) forms of communication (TV commercials among them) has the ability to influence habits of thought. If he did not assume this--if we to were to take it away (ie, deny it)--then the argument falls apart! But it is not sufficient. If we plugged it in, the conclusion does not for sure follow because we don't know if TV commercials count as "certain forms".
Last edited by Testluv on Wed Nov 11, 2009 11:04 pm, edited 1 time in total.
Kaplan Teacher in Toronto

Legendary Member
Posts: 869
Joined: Wed Aug 26, 2009 3:49 pm
Location: California
Thanked: 13 times
Followed by:3 members

by heshamelaziry » Wed Nov 11, 2009 10:49 pm
I can't thank you enough :D . This was great explanation.

By the way, just for sharing, the answer choice is an assumption that eliminates another reason that could have mad the residents unable to receive the radio signal.

I think when i am faced with question with more than two things going on, I get confused. In a straight forward, short question, i do better.

One last question: In flaw questions, what kind of assumption is usually asked for ?

GMAT Instructor
Posts: 1302
Joined: Mon Oct 19, 2009 2:13 pm
Location: Toronto
Thanked: 539 times
Followed by:164 members
GMAT Score:800

by Testluv » Wed Nov 11, 2009 11:08 pm
heshamelaziry wrote:I can't thank you enough :D . This was great explanation.

By the way, just for sharing, the answer choice is an assumption that eliminates another reason that could have mad the residents unable to receive the radio signal.

I think when i am faced with question with more than two things going on, I get confused. In a straight forward, short question, i do better.

One last question: In flaw questions, what kind of assumption is usually asked for ?
A flaw question is asking for a reasoning mistake. Whenever the arguer takes something for granted (ie, assumes something), the arguer has committed a mistake in her reasoning. So, whenver the argument has an assumption, the arguer's reasoning is flawed; and, whenever the arguer's reasoning is flawed, she is making an assumption. They are like two sides of the same coin.

That said, the flaw being asked for is usually related to a necessary rather than a sufficient assumption.
Kaplan Teacher in Toronto

Master | Next Rank: 500 Posts
Posts: 168
Joined: Mon Apr 13, 2009 6:48 pm
Thanked: 6 times

by punitkaur » Thu Nov 12, 2009 12:07 pm
Hi Testluv,

I find this question really hard and I am still trying to figure out how the negation of first choice weakens the conclusion.

I read your explanation. Although you explained it in detail I am not able to relate the assumption that " ALL of the radios were always receiving radio signals" to the first choice listed.

A)Few if any of the RDS radio stations that began broadcasting in Verdland after 1994 broadcast to people with RDS-equipped radios living in areas not previously reached by RDS stations.

the first choice says, that between 1994 and 1996, the radio stations were broadcasting to people not previously reachable, that means more people. how does this relate to the assumption that all of radios were always receiving radio signals?

I am completely confused :(

Master | Next Rank: 500 Posts
Posts: 202
Joined: Tue Sep 08, 2009 11:34 pm
Thanked: 15 times
GMAT Score:760

by cbenk121 » Thu Nov 12, 2009 12:19 pm
punitkaur wrote:Hi Testluv,

I find this question really hard and I am still trying to figure out how the negation of first choice weakens the conclusion.

I read your explanation. Although you explained it in detail I am not able to relate the assumption that " ALL of the radios were always receiving radio signals" to the first choice listed.

A)Few if any of the RDS radio stations that began broadcasting in Verdland after 1994 broadcast to people with RDS-equipped radios living in areas not previously reached by RDS stations.

the first choice says, that between 1994 and 1996, the radio stations were broadcasting to people not previously reachable, that means more people. how does this relate to the assumption that all of radios were always receiving radio signals?

I am completely confused :(
What's the logical opposite of none? You could say "some", but that's not as clear as saying "all".

ALL of the radio stations that began broadcasting to people with RDS equipped radios living in areas not previously reached.

IF this is true, then that means the number of listeners has GONE UP, even with the # of radios static. All the other choices, when negated, either strengthen the conclusion or suggest the listener total WENT DOWN, which doesn't contradict his argument, which is simply that the count did not GO UP (includes staying static or going down).

GMAT Instructor
Posts: 1302
Joined: Mon Oct 19, 2009 2:13 pm
Location: Toronto
Thanked: 539 times
Followed by:164 members
GMAT Score:800

by Testluv » Thu Nov 12, 2009 1:39 pm
cbenk121 wrote:
punitkaur wrote:Hi Testluv,

I find this question really hard and I am still trying to figure out how the negation of first choice weakens the conclusion.

I read your explanation. Although you explained it in detail I am not able to relate the assumption that " ALL of the radios were always receiving radio signals" to the first choice listed.

A)Few if any of the RDS radio stations that began broadcasting in Verdland after 1994 broadcast to people with RDS-equipped radios living in areas not previously reached by RDS stations.

the first choice says, that between 1994 and 1996, the radio stations were broadcasting to people not previously reachable, that means more people. how does this relate to the assumption that all of radios were always receiving radio signals?

I am completely confused :(
What's the logical opposite of none? You could say "some", but that's not as clear as saying "all".

ALL of the radio stations that began broadcasting to people with RDS equipped radios living in areas not previously reached.

IF this is true, then that means the number of listeners has GONE UP, even with the # of radios static. All the other choices, when negated, either strengthen the conclusion or suggest the listener total WENT DOWN, which doesn't contradict his argument, which is simply that the count did not GO UP (includes staying static or going down).
The logical opposite of "none" is "some". "none" means none; "some" means at least one. Having at least one is the opposite of having none.

Choice A is difficult to work with. Better off identifying assumption BEFORE going to the choices (the way I did above). You can use denial test to verify whether the assumption you've identified is correct. Then, it is easier to see that A matches.

But, if we are going to deny an answer choice, we have to deny the meaning of the choice. The choice can only mean one thing (test-maker can't use it if there were multiple valid interpretations). Therefore, there is only one true denial or opposite of an answer choice. Because the choice starts with "few, if any", a good denial would be "many".

So, the denial of choice A would be:

If after 1994, MANY of the radio stations' signals WERE broadcasting to people who previously did NOT receive the signals, then after the increase in radio stations, more people will be receiving the signals, and the argument falls apart.

So, the denial here was changing "few, if any" to "many."

If you think there are multiple linguistic places that you can deny it, then you should think about what the choices MEANS, and deny it at its meaning. If you're not sure what it means and you really want to use the denial test, then you have to deny at each place systematically (inefficient). You don't want to deny twice at the same time because denying twice (not, not) returns you to the original
Last edited by Testluv on Thu Nov 12, 2009 7:32 pm, edited 1 time in total.
Kaplan Teacher in Toronto

Legendary Member
Posts: 869
Joined: Wed Aug 26, 2009 3:49 pm
Location: California
Thanked: 13 times
Followed by:3 members

by heshamelaziry » Thu Nov 12, 2009 6:22 pm
punitkaur wrote:Hi Testluv,
Last edited by heshamelaziry on Thu Nov 12, 2009 6:45 pm, edited 1 time in total.

Legendary Member
Posts: 869
Joined: Wed Aug 26, 2009 3:49 pm
Location: California
Thanked: 13 times
Followed by:3 members

by heshamelaziry » Thu Nov 12, 2009 6:27 pm
Wonder if anyone could give a brief explanation of the other answer choices, besides "A" ?

Master | Next Rank: 500 Posts
Posts: 202
Joined: Tue Sep 08, 2009 11:34 pm
Thanked: 15 times
GMAT Score:760

by cbenk121 » Thu Nov 12, 2009 7:07 pm
heshamelaziry wrote:Wonder if anyone could give a brief explanation of the other answer choices, besides "A" ?
(B) In 1996 most Verdlanders who lived within the listening area of a RDS station already had a radio equipped to receive RDS.

-->If this is not true, then few people have a radio as of 1996. This doesn't attack the argument, because it doesn't show a way that listenership could've increased.
(C) Equipping a radio station with RDS technology does not decrease the station's listening area.

-->If this is not true, then the station's listening area would decrease, providing a way that the number of stations could have increased without increasing number of listeners. This STRENGTHENS.

(D) In 1996 Verdlanders who did not own radios equipped to receive RDS could not receive any programming from the RDS radio stations that began broadcasting in Verdland after 1994.

-->If this is not true, then Verdlanders who owned RDS radios could not receive programming from RDS radio stations. This STRENGTHENS. If you happened to negate the second "not", you'd get "Verdlanders who did not own RDS radios could receive any RDS programming", which directly contradicts a premise in the stimulus...which I believe is a no-no?

(E) The RDS radio stations in Verdland in 1996 did not all offer the same type of programming.

-->This has nothing to do with the argument.